{ "cells": [ { "cell_type": "code", "execution_count": 41, "metadata": {}, "outputs": [], "source": [ "import numpy as np\n", "import matplotlib.pyplot as plt\n", "\n", "#CONSTANTES\n", "\n", "#Nombre d'Avogadro\n", "avo=6.02214076e23\n", "\n", "#Constante de Boltzmann\n", "k=1.38064852e-23 #m2 kg s-2 K-1 ou J/K\n", "T=298.13\n", "kT=k*T #énergie thermique par molécule\n", "RGP=k*avo\n", "\n", "#Charge de l'électron\n", "e=1.602e-19\n", "F=e*avo\n", "\n", "#Permittivité électrique de l'eau\n", "eps=8.85418782e-12*78.5 #m-3 kg-1 s4 A2\n", "\n", "#Viscosité (dynamique) de l'eau à 20°C\n", "mu=1e-3 #Pa.s" ] }, { "cell_type": "markdown", "metadata": {}, "source": [ "# 1 Flux électro-osmotique dans une membrane d’électrodialyse\n", "Des membranes d’électrodialyse d’une épaisseur de 500 m ayant un potentiel de surface de -60 mV sont utilisées dans un stack d’ED avec un champ électrique de 700 V/m. Le rayon des pores de la membrane est de 0,1 m en moyenne. La porosité de surface est de 10 %.\n", "\n", "1)\tCalculer le débit électro-osmotique à travers une membrane de 20 cm de long et 20 cm de large.\n", "\n", ">Si on considère que la longueur de Debye est petite devant le diamètre des pores de la membranes, on peut appliquer l'équation d'Helmholtz-Smoluchowski pour relier la vitesse electroosmotique au potentiel zeta selon $$u_{EO}=-\\frac{\\epsilon\\zeta E}{\\mu}$$\n", "Le débit à travers la membrane est déterminé par le produit de la vitesse par la section des pores. La sections des pores est le produit de la porosité de surface de la membrane et de la surface de la membrane :\n", "$$Q_{EO}=u_{EO}\\epsilon S$$\n", "Le signe positif signifie que le flux d'eau est dans la direction du champ électrique (c'est à dire vers la cathode). Ceci est normal car la charge de la membrane est négative (membrane anionique et donc membrane échangeuse de cation) et donc les cations qui s'accumulent à la surface des pores sont transportés vers la cathode (négative).\n", "\n", "2)\tDéterminer la surpression à appliquer afin de compenser le débit électro-osmotique par un débit convectif inverse.\n", "\n", ">Le débit électro-osmotique conduit à diluer le concentrat. Il va donc géner la concentration https://fr.wikipedia.org/wiki/%C3%89lectrodialyse#/media/Fichier:Electrodialysis.jpg\n", "Pour contrer le flux electro-osmotique on peut exercer une surpression dans le concentrat. La difference de pression entre concentrat et diluat peut contrer le flux electro-osmotique. La convection forcée à travers un pore peut se décrire avec la loi de Poiseuille\n", "$$u_{moy}=\\frac{R^2}{8\\mu}\\frac{\\Delta P}{L}$$\n" ] }, { "cell_type": "code", "execution_count": 42, "metadata": {}, "outputs": [ { "name": "stdout", "output_type": "stream", "text": [ "La vitesse electro-osmotique est de 2.919e-05 m/s\n", "Le débit electro-osmotique est de 1.168e-07 m3/s\n", " correspondant à 0.420 L h-1\n", "La contre pression à appliquer est de 1.168e+04 Pa\n", " soit 0.117 bar\n" ] } ], "source": [ "E=700 #V/m\n", "zeta=-0.06 #V\n", "L=5e-4 #m\n", "R=1e-7 #m\n", "\n", "U_EO=-(eps*zeta*E/mu)\n", "print('La vitesse electro-osmotique est de {0:.3e} m/s'.format(U_EO))\n", "porosite=0.1\n", "S=0.2*0.2\n", "Q_EO=U_EO*porosite*S\n", "print('Le débit electro-osmotique est de {0:.3e} m3/s'.format(Q_EO))\n", "print(' correspondant à {0:.3f} L h-1'.format(Q_EO*1000*3600))\n", "\n", "DP=U_EO*8*mu*L/(R**2)\n", "print('La contre pression à appliquer est de {0:.3e} Pa'.format(DP))\n", "print(' soit {0:.3f} bar'.format(DP*1e-5))" ] }, { "cell_type": "markdown", "metadata": {}, "source": [ "# 2-Analyse par micro-électrophorèse \n", "Un tube cylindrique d’un rayon de 5 mm et de 10 cm de long est fermé aux deux bouts par des électrodes auxquelles est appliquée une différence de potentiel de 10 V. Le tube contient une solution aqueuse d’un sel complètement dissocié symétrique de valence 2 à une concentration de 1 mol.m-3. La paroi du tube possède une charge de surface de 1,43 mV. La température est de 25°C.\n", "\n", "1)\tDéterminer la longueur de Debye sur la paroi du tube.\n", "\n", ">La longueur de Debye qui caractérise la portée de la douche couche electrostatique s'écrit : \n", "$$\\lambda_D=\\sqrt{\\frac{\\epsilon RT}{2 F^2 \\sum_i{z_i^2c_i}}}$$\n", "avec la force ionique définit selon :\n", "$$I=\\frac{1}{2}\\sum_i{z_i^2c_i}$$\n", "La longueur de Debye est beaucoup plus petite que la taille du capillaire.\n", "\n", "2)\tDéterminer :\t\n", "- la distance radiale à laquelle la vitesse du fluide est nulle \n", "- la vitesse du fluide au centre du tube\n", "- la vitesse du fluide à mi-distance entre la paroi et les plans stationnaires\n", "\n", ">La vitesse electro-osmotique peut se calculer selon : \n", "$$u_{EO}=-\\frac{\\epsilon\\zeta E}{\\mu}$$\n", "Comme le potentiel zeta est positif, la vitesse electro-osmotique (à la paroi du tube) est négative car opposée à la direction du champ électrique (dirigée vers l'anode). Le profil de vitesse dans le tube est donné par un couplage entre une convection forcée et la vitesse électro-osmotique :\n", "$$u=u_{EO}(1-2(1-\\frac{r^2}{a^2}))$$\n", "\n", "\n", "3)\tCalculer la vitesse au niveau des plans stationnaires de particule de latex polystyrene d’un rayon de 0.1 micrometre et portant 0.001 electrons par nm3.\n", "\n", ">Le calcul de la vitesse électro-phorétique dépend du rapport de la longueur de Debye sur la taille de la particule. Ici la longueur de Debye est petite devant la taille des particules. On peut donc déduire la vitesse de l'équation d'Helmholtz-Smoluchowski :\n", "$$u_{EO}=\\frac{\\epsilon\\zeta E}{\\mu}$$\n", "Le potentiel zeta de la particule peut s'estimer à partir de la charge avec l'équation suivante :\n", "$$q=4\\pi\\epsilon a(1+\\frac{a}{\\lambda_D})\\zeta$$\n", "\n", "4) Calculer la concentration critique de coagulation des particules de latex. Est-ce que les particules sont agrégés dans les conditions de la microelectrophorèse ?\n", "\n", "> La concentration critique de coagulation peut se calculer avec la loi de Schlutze Hardy :\n", "$$ccc=3.8 10^{-36} \\frac{\\Upsilon_0^4}{A^2z^6}$$\n", "avec \n", "$$\\Upsilon_0=tanh(\\frac{ze\\zeta}{4k_BT})$$\n" ] }, { "cell_type": "code", "execution_count": 43, "metadata": {}, "outputs": [ { "name": "stdout", "output_type": "stream", "text": [ "La force ionique de la solution est de 0.004 M\n", "La longueur de Debye est de 4.810e-09 m\n", "La vitesse electro-osmotique est de -1.029e-07 m/s\n", "Les plans stationnaires sont situés à r= 3.536e-03 m du centre\n", "Vitesse du fluide au centre du tube : 1.029e-07 m/s\n", "Vitesse du fluide à mi-distance : 5.143e-08 m/s\n", "Vitesse du fluide aux plans stationnaires : 2.284e-23 m/s\n", "Vitesse du fluide à la paroi : -1.029e-07 m/s\n", "Charge des particules : 6.710e-16 C\n", "Potentiel zeta des particules : 3.526e-02 V\n", "Vitesse electro-phorétique : 2.451e-06 m/s\n", "Concentration critique de coagulation en sel divalent 4.419e+01 mol/m3\n", "Concentration critique de coagulation en sel divalent 4.419e-02 mol/L\n" ] }, { "data": { "image/png": "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\n", "text/plain": [ "
" ] }, "metadata": { "needs_background": "light" }, "output_type": "display_data" } ], "source": [ "z=2\n", "c=1 #mol/m3\n", "I=0.5*(z**2*c+z**2*c)/1000 #mol/L\n", "print ('La force ionique de la solution est de', I, ' M')\n", "Ld=np.sqrt(eps*RGP*T/(2*F*F*I*1000))\n", "print ('La longueur de Debye est de {0:.3e}'.format(Ld), 'm')\n", "\n", "#Calcul de la vitesse d'électro-osmose\n", "zeta=1.48e-3\n", "a=5e-3\n", "E=10/0.1 #V/m\n", "U_EO=-(eps*zeta*E/mu)\n", "print('La vitesse electro-osmotique est de {0:.3e} m/s'.format(U_EO))\n", "\n", "#Calcul du profil de vitesse du fluide induit par l'éectro-osmose\n", "def u(r):\n", " u=U_EO*(1-2*(1-r**2/a**2))\n", " return u\n", "\n", "rstat=a/np.sqrt(2)\n", "print ('Les plans stationnaires sont situés à r= {0:.3e} m du centre'.format(a/np.sqrt(2)))\n", "print ('Vitesse du fluide au centre du tube : {0:.3e} m/s'.format(u(0)))\n", "print ('Vitesse du fluide à mi-distance : {0:.3e} m/s'.format(u(a/2)))\n", "print ('Vitesse du fluide aux plans stationnaires : {0:.3e} m/s'.format(u(a/np.sqrt(2))))\n", "print ('Vitesse du fluide à la paroi : {0:.3e} m/s'.format(u(a)))\n", "\n", "rtab=np.linspace(-a,a,100)\n", "plt.plot(rtab,u(rtab))\n", "plt.plot([-a,a],[0,0], '--k')\n", "plt.plot([-a,-a],[-U_EO,U_EO], 'k')\n", "plt.plot([a,a],[-U_EO,U_EO], 'k')\n", "plt.plot([-rstat,-rstat],[-U_EO,U_EO], '--r')\n", "plt.plot([rstat,rstat],[-U_EO,U_EO], '--r')\n", "plt.text(-rstat+0.0001,U_EO/2,'plan stationnaire',rotation=90, color='r')\n", "plt.text(rstat+0.0001,U_EO/2,'plan stationnaire',rotation=90, color='r')\n", "plt.title('Profil de vitesse du fluide induit par l\\'électro-osmose')\n", "plt.xlabel('Distance au centre, r (m)')\n", "plt.ylabel('Vitesse du fluide, u (m/s)')\n", "\n", "#Calcul de la vitesse d'electrophorese\n", "nelec=0.001 #electron par nm3\n", "R=1e-7 #m\n", "Vp=4*np.pi*(R**3)/3\n", "q=nelec*e*Vp*(1e9**3) #C\n", "print ('Charge des particules : {0:.3e} C'.format(q)) \n", "zeta=q/(4*np.pi*eps*R*(1+R/Ld))\n", "print ('Potentiel zeta des particules : {0:.3e} V'.format(zeta))\n", "U_EP=(eps*zeta*E/mu)\n", "print ('Vitesse electro-phorétique : {0:.3e} m/s'.format(U_EP))\n", "\n", "#Calcul de la ccc\n", "Hamaker=1.3e-20 #J\n", "upsilon=np.tanh(z*e*zeta/(4*kT))\n", "ccc=3.8e-36*(upsilon**4/(Hamaker**2*z**6))\n", "print ('Concentration critique de coagulation en sel divalent {0:.3e} mol/m3'.format(ccc))\n", "print ('Concentration critique de coagulation en sel divalent {0:.3e} mol/L'.format(ccc/1000))" ] }, { "cell_type": "markdown", "metadata": {}, "source": [ "# 3-Electrophorèse continue\n" ] }, { "cell_type": "markdown", "metadata": {}, "source": [ "Une électrophorèse est réalisée au sein d’une veine liquide parallélépipédique de 30 cm de longueur (Lx), 5 cm de largeur (Ly), et 2 mm d’épaisseur (Lz),. On injecte dans la veine liquide un échantillon de largeur lc=0,5 mm contenant un mélange de deux protéines (la serum albumine bovine (SAB) et l’-lactalbumine) avec un débit Q de 0,5 l/h. En sortie de la veine liquide, on récupère l’écoulement dans une série de petits capillaires d’un diamètre de 0,06 mm. On applique, parallèlement à l’écoulement, une différence de potentiel de 24 V. \n", "\n", "Données : \t\n", "Mélange de protéines\t\t\n", "- Force ionique du mélange 10-3 M KCl\n", "- SAB : mobilité –15 10-9 m2.V-1.s-1, \t\t\t\t\n", "- Hémoglobine : mobilité -22.10-9 m2.V-1.s-1 \t\t\t\n", "- Coefficient de diffusion des deux protéines : D=6.10-11 m2/s\n", "- Rayon des protéines : R=4 nm\n", "- Constante de Hamaker pour les protéines : A=5.10-20 J\n", "\n", "Paroi de la cellule d’électrophorèse\n", "- Charge de surface des parois de la cellule : -10 mV\n", "\n", "1)\tDéterminer l’intensité de la vitesse électrophorétique pour les deux protéines.\n" ] }, { "cell_type": "code", "execution_count": 44, "metadata": {}, "outputs": [ { "name": "stdout", "output_type": "stream", "text": [ "Vitesse SAB : -7.200e-06 m/s\n", "Vitesse HM : -1.056e-05 m/s\n" ] } ], "source": [ "mSAB=-15e-9\n", "mHM=-22e-9\n", "E=24/0.05 #V/m\n", "uSAB=mSAB*E\n", "uHM=mHM*E\n", "print ('Vitesse SAB : {0:.3e} m/s'.format(uSAB)) \n", "print ('Vitesse HM : {0:.3e} m/s'.format(uHM)) " ] }, { "cell_type": "markdown", "metadata": {}, "source": [ "2)\tCette vitesse dans la direction y se combine à l ‘écoulement du liquide suivant x. Déterminer la distance, Y, qui est parcourue suivant y en sortie de la veine liquide par les deux protéines et en déduire la distance séparant les deux protéines en sortie, $\\Delta Y$.\n", "\n", ">Le temps de séjour des protéines dans la cellule est le rapport du volume de la cellule sur le débit volumique. La déviation de la trajectoire des protéines sera alors donné par le produit de la vitesse selon Y par le temps de séjour. \n", "$$Y_x=u_x*t_s$$\n", "$$\\Delta Y=\\lvert Y_{SAB}-Y_{HM} \\rvert$$" ] }, { "cell_type": "code", "execution_count": 45, "metadata": {}, "outputs": [ { "name": "stdout", "output_type": "stream", "text": [ "Temps de sejour 216.00000000000003 s\n", "Difference de déviation 0.0007257599999999999 m\n" ] } ], "source": [ "Q=0.5 #L/h\n", "V=0.3*0.05*2e-3 #m3\n", "t_s=V/(Q*0.001/3600)\n", "print ('Temps de sejour ',t_s, ' s')\n", "DY=(uSAB-uHM)*t_s\n", "print ('Difference de déviation ',DY, ' m')" ] }, { "cell_type": "markdown", "metadata": {}, "source": [ "3)\tEn considérant les espèces séparées si Y en sortie de veine liquide est supérieur au diamètre des capillaires, la séparation des deux espèces a-t-elle lieue ?\n", "\n", "> Oui car la différence de déviation est bien plus grande que la taille des capillaires recupérant la veine liquide en sortie\n", "\n", "\n" ] }, { "cell_type": "markdown", "metadata": {}, "source": [ "4)\tCalculer la vitesse électro-osmotique, uEO, due à la charge de surface de la paroi de la veine liquide." ] }, { "cell_type": "code", "execution_count": 46, "metadata": {}, "outputs": [ { "name": "stdout", "output_type": "stream", "text": [ "La vitesse electro-osmotique est de 3.336e-06 m/s\n" ] } ], "source": [ "zeta_w=-0.01\n", "U_EO=-(eps*zeta_w*E/mu)\n", "print('La vitesse electro-osmotique est de {0:.3e} m/s'.format(U_EO))" ] }, { "cell_type": "markdown", "metadata": {}, "source": [ "6) La vitesse du fluide selon y, $v_y$, dans la veine est due à une combinaison de la vitesse électro-osmotique proche des parois perpendiculaires aux électrodes et à une vitesse de convection forcée. \n", "$$v_y=v_{EO}+v_{CF}$$\n", "La convection forcée dans une veine parrallélépipédique s'écrit :\n", "$$v_{CF}=v_{max}(1-\\frac{z^2}{d^2})$$\n", "où v_{max} est une constante qui dépend notamment de la pression appliquée. Dans notre cas, nous ne connaissons pas cette pression mais par contre nous savons que le débit de liquide dans la direction y doit être nul (fluide incompressible et volume constant). Cette condition s'écrit :\n", "$$Q \\propto \\int_{0}^{d} v_{y}(z) \\, \\mathrm{d}z\\mathrm{d}y=0$$\n", "Pour satisfaire cette équation il faut que :\n", "$$\\int_{0}^{d} (v_{EO}+v_{max}(1-\\frac{z^2}{d^2})) \\, \\mathrm{d}z=0$$\n", "soit après intégration :\n", "$$v_{max}=-\\frac{3}{2}v_{EO}$$\n", "La vitesse du fluide selon y suivra la relation :\n", "$$v_y=\\frac{v_{EO}}{2}(3\\frac{z^2}{d^2}-1)$$\n", "Avec une vitesse égale à $-\\frac{v_{EO}}{2}$ au centre de la veine et une vitesse $v_{EO}$ à la paroi." ] }, { "cell_type": "code", "execution_count": 47, "metadata": {}, "outputs": [ { "name": "stdout", "output_type": "stream", "text": [ "vy en z=0 : -1.668128985288e-06\n", "vy en z=0.25 mm : -1.3553548005465e-06\n", "vy en z=d : 3.336257970576e-06\n" ] }, { "name": "stderr", "output_type": "stream", "text": [ "C:\\Users\\patri\\Anaconda3\\lib\\site-packages\\ipykernel_launcher.py:13: RuntimeWarning: invalid value encountered in sqrt\n", " del sys.path[0]\n", "C:\\Users\\patri\\Anaconda3\\lib\\site-packages\\ipykernel_launcher.py:14: RuntimeWarning: invalid value encountered in sqrt\n", " \n" ] }, { "data": { "image/png": "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\n", "text/plain": [ "
" ] }, "metadata": { "needs_background": "light" }, "output_type": "display_data" } ], "source": [ "d=1e-3 #mm\n", "dinj=0.25e-3\n", "\n", "def vy(z):\n", " return U_EO*(3*(z**2)/(d**2)-1)/2\n", "\n", "print ('vy en z=0 :', vy(0))\n", "print ('vy en z=0.25 mm :', vy(dinj))\n", "print ('vy en z=d :', vy(d))\n", "\n", "zplot=np.linspace(0,d,500)\n", "plt.plot(zplot, vy(zplot))\n", "plt.plot(zplot, 0.01*(dinj**2-zplot**2)**(0.5), 'r' )\n", "plt.plot(zplot, -0.01*(dinj**2-zplot**2)**(0.5), 'r' )\n", "plt.text(0, 0, 'injecteur' )\n", "plt.text(d, 0, 'paroi' )\n", "plt.show()" ] }, { "attachments": { "UEO.jpg": { "image/jpeg": "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" } }, "cell_type": "markdown", "metadata": {}, "source": [ "7)\n", "![UEO.jpg](attachment:UEO.jpg)\n", "\n", "8) Cette différence de vitesse va deviée les protéines différemment selon leur position dans l injecteur. La tâche de protéine en sortie aura une forme de croissant. La différence de position en sortie sera liée à la différence de vitesse fois le temps de séjour.\n", "$$\\Delta Y_{EO}=t_s \\Delta u_{EO}$$" ] }, { "cell_type": "code", "execution_count": 48, "metadata": {}, "outputs": [ { "name": "stdout", "output_type": "stream", "text": [ "Difference de déviation EO 6.7559223904164e-05 m\n", "Pourcentage 9.30875549825893 %\n" ] } ], "source": [ "DYEO=(vy(dinj)-vy(0))*t_s\n", "print ('Difference de déviation EO',DYEO, ' m')\n", "print ('Pourcentage ',100*DYEO/DY, '%')" ] }, { "cell_type": "markdown", "metadata": {}, "source": [ "9) La diffusion peut aussi générer une dispersion autour de la position initiale qui peut être estimée par : \n", "$$\\Delta Y_{D}=4 \\sqrt{2Dt}$$" ] }, { "cell_type": "code", "execution_count": 49, "metadata": {}, "outputs": [ { "name": "stdout", "output_type": "stream", "text": [ "Dispersion par diffusion 0.0006439875775199395 m\n" ] } ], "source": [ "D=6e-11 #m2/s\n", "DYD=4*(2*D*t_s)**0.5\n", "print ('Dispersion par diffusion',DYD, ' m')" ] }, { "attachments": { "sortie2.jpg": { "image/jpeg": "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" } }, "cell_type": "markdown", "metadata": {}, "source": [ "10) La séparation des protéines semble possible. Les filets de protéines devraient être récupérés des capillaires différents mais les protéines risquent de ne pas être complètements pures. La diffusion va notamment étaler les tâches en sortie de façon significative. \n", "![sortie2.jpg](attachment:sortie2.jpg)\n", "\n", "Par ailleurs on considère dans ces calculs l absence de convection naturelle due à l échauffement du fluide par effet Joules : nous nous sommes mis dans les conditions d une séparation réalisée en station interntionale c est à dire à 0 g !" ] }, { "cell_type": "code", "execution_count": null, "metadata": {}, "outputs": [], "source": [] } ], "metadata": { "kernelspec": { "display_name": "Python 3", "language": "python", "name": "python3" }, "language_info": { "codemirror_mode": { "name": "ipython", "version": 3 }, "file_extension": ".py", "mimetype": "text/x-python", "name": "python", "nbconvert_exporter": "python", "pygments_lexer": "ipython3", "version": "3.7.3" } }, "nbformat": 4, "nbformat_minor": 2 }